PT12.S1.Q05 - oscar: I have been accused of plagiarizing

The 180 Bro_OVOThe 180 Bro_OVO Alum Inactive ⭐
edited January 2017 in Logical Reasoning 1392 karma
So, I've done this question many many times. And I've never felt great about it. Can some run it down for me?
Specifically, answer choice A.
Here's what A looks like:
A:
Writer has right ---> Author granted writer the right

I believe A is incorrect because "Writer has right" should not be in the sufficient condition.
Rather, to be correct, A should look like this:

Author granted writer the right ---> Writer has right


Is this accurate?


(if you want to add anything else helpful, it'd be greatly appreciated).

Comments

  • BinghamtonDaveBinghamtonDave Alum Member 🍌🍌
    edited January 2017 8689 karma
    These questions are essentially sufficient assumption questions. We are given a set of facts with a conclusion and we are asked to supply something that would allow the conclusion to follow, the best we can. Modern variations of these question stems invariably say "most helps to justify," but this question does not. It is paramount in these questions to identify the conclusion. For the longest time when I first started studying for the LSAT I had difficulty with these problems because I found the arguments so disjointed that they confused me.


    Oscar calls the claim of plagiarizing "unwarranted." What is Oscar's evidence for calling the claim "unwarranted?" That the writer gave Oscar permission in private. For this question, we are looking for something that in a nut shell says that if we have permission from the author then it isn't plagiarism. Your assertion about (A) is entirely correct. It isn't what we need.
  • BinghamtonDaveBinghamtonDave Alum Member 🍌🍌
    8689 karma
    I should also state that even on PT 12, the LR section follows a familiar pattern, we have other people's opinion, followed by a denial of that opinion, followed by a concession that the action claimed at bottom by the other people's opinion did indeed occur, but it is qualified by a statement that is supposed to mitigate the extent of other people's opinion: the final sentence. Consider an analogous argument which has a form that could show up on contemporary exams:

    Some people say that Tom Brady is the greatest quarterback of all time. They are wrong. Although Tom Brady has been great for more than 15 years and is headed to his 7th Super Bowl, he is 4th on the all time passing yards list.

    If the LSAT had provided us with this argument in the form of a pseudo sufficient assumption/ sufficient assumption, what would be the SA? In other words, how I look at these questions is to ask myself: ok, so what are you saying? What I am saying is that if you are 4th on that list then you aren't the greatest. Because the English language is sometimes so complex, there are numerous variations on this SA, such as "if you aren't #1 on that list then you are not the greatest of all time."

    Hope this helps,
    David

  • SamiSami Live Member Sage 7Sage Tutor
    edited January 2017 10774 karma
    @"The 180 Bro_OVO" said:

    I believe A is incorrect because "Writer has right" should not be in the sufficient condition.
    Rather, to be correct, A should look like this:

    Author granted writer the right ---> Writer has right

    I would actually agree with your method here. That's what I did to quickly eliminate A. We need a principle that helps us conclude about author having "the right to publish". But answer choice "A" will only allow us to conclude that either the "author did not have the right" or "author granted the right". None of which match what we are trying to conclude; that author did have that right.

    Answer choice "B" starts about a scenario about what is the case when "passages are more than few sentences long". We need it to be about a scenario about "giving permission". So this is wrong.

    Answer choice "C" is about what we can conclude when an "article has not been published". We don't care what happens when an article is not published. We need to know about a scenario where "you have permission" and we can conclude "its not plagiarism".

    Answer choice "D" is talking about what we can conclude "if an author has relinquished his right to material". In Principle question, my immediate question is "would an author giving permission be considered the same as relinquishing right to material". It would! now let's look at the conclusion. Answer choice "D" concludes, "an author is entitled to quote freely without attribution". So would it be plagiarism if the author is entitled to quote freely without attribution? Nope. So we are definitely concluding "this is not plagiarism".

    I think the trick to these types of question is to remember that we will not get our exact wording or what we are concluding in our stimulus but we are going to get an answer choice phrased in other scenarios which should still help us to conclude what our stimulus concludes.

    Answer choice "E"is about what an author is allowed to do if he has published the same thing in another one of his own works. This scenario is useless to us as well because we have a situation in our stimulus about an author quoting someone else's work not his own.
Sign In or Register to comment.